2021 AMC 10B Problems/Problem 19

Revision as of 02:49, 12 February 2021 by Etvat (talk | contribs)

Problem

Suppose that $S$ is a finite set of positive integers. If the greatest integer in $S$ is removed from $S$, then the average value (arithmetic mean) of the integers remaining is $32$. If the least integer is $S$ is [i]also[/i] removed, then the average value of the integers remaining is $35$. If the greatest integer is then returned to the set, the average value of the integers rises of $40$. The greatest integer in the original set $S$ is $72$ greater than the least integer in $S$. What is the average value of all the integers in the set $S ?$

$\textbf{(A)} ~36.2 \qquad\textbf{(B)} ~36.4 \qquad\textbf{(C)} ~36.6 \qquad\textbf{(D)} ~36.8 \qquad\textbf{(E)} ~37$

Solution 1

Let the lowest value be L and the highest G, and let the sum be Z and the amount of numbers n. We have $\frac{Z-G}{n-1}=32$, $\frac{Z-L-G}{n-2}=35$, $\frac{Z-L}{n-1}=40$, and $G=L+72$. Clearing denominators gives $Z-G=32n-32$, $Z-L-G=35n-70$, and $Z-L=40n-40$. We use $G=L+72$ to turn the first equation into $Z-L=32n+40$, which gives $n=10$. Turning the second into $Z-2L=35n+2$ we see $L=8$ and $Z=368$ so the average is $\frac{Z}{n}=\boxed{(D)36.8}$ ~aop2014

Solution 2 (much more thorough version of Sol 1)

Let $S = {x_1, x_2, \dots, x_n}$ in increasing order. There are $n$ integers in this set.

The greatest integer in $S$ is $x_n$, so we need to remove this and get the average as $32$. There are $n-1$ total terms. Thus, \[\frac{x_1+x_2+\dots+x_{n-1}}{n-1} = 32\]\[x_1+x_2+\dots+x_{n-1} = 32(n-1).\]

The least integer is $x_1.$ Removing it, we have \[x_2+\dots+x_{n-1} = 35(n-2).\]

Finally, adding back $x_n$ we have \[x_2+\dots+x_n = 40(n-1).\]

Since the greatest integer in the original set $S$ is $72$ greater than the least integer in $S$, we have \[x_n = x_1 + 72.\]

Substituting this into the third equation, we have $x_2 + \dots + x_{n-1} + x_1 + 72 = 40(n-1).$ Rearranging, this becomes $x_1 + \dots + x_{n-1} = 40(n-1) - 72.$

But the LHS of this equation is the same as the LHS of the first equation! Equating them, we have $40(n-1)-72 = 32(n-1)$, which means $n = 10.$

Let's plug $n = 10$ into all of our previous equations. We have the following system: \[x_1+x_2+\dots+x_9 = 288\]\[x_2+\dots+x_9 = 280\]\[x_2+\dots+x_{10} = 360\]

We're asked to find the average of $S$, which is $\frac{x_1+x_2+\dots+x_{10}}{10}.$ Let's focus on the numerator. We can find $x_1$ by subtracting the second equation from the first, to get $x_1 = 8.$ We can also find $x_{10}$ by subtracting the second equation from the third, to get $x_{10} = 80.$

We also know $x_2+\dots+x_9$ to equal $280$. So when we sum up our three values, we get \[x_1 + (x_2 + \dots + x_9) + x_{10} = 8 + 280 + 80 = 368.\]

Therefore, the sum of the values is $368$, and finding the average in $10$ terms we have \[\frac{368}{10} = \boxed{(D)\text{ }36.8}.\]

-PureSwag

2021 AMC 10B (ProblemsAnswer KeyResources)
Preceded by
Problem 18
Followed by
Problem 20
1 2 3 4 5 6 7 8 9 10 11 12 13 14 15 16 17 18 19 20 21 22 23 24 25
All AMC 10 Problems and Solutions